Diễn Đàn MathScopeDiễn Đàn MathScope
  Diễn Đàn MathScope
Ghi Danh Hỏi/Ðáp Community Lịch

Go Back   Diễn Đàn MathScope > Sơ Cấp > Chuyên Đề Mathscope Chọn Lọc

News & Announcements

Ngoài một số quy định đã được nêu trong phần Quy định của Ghi Danh , mọi người tranh thủ bỏ ra 5 phút để đọc thêm một số Quy định sau để khỏi bị treo nick ở MathScope nhé !

* Nội quy MathScope.Org

* Một số quy định chung !

* Quy định về việc viết bài trong diễn đàn MathScope

* Nếu bạn muốn gia nhập đội ngũ BQT thì vui lòng tham gia tại đây

* Những câu hỏi thường gặp

* Về việc viết bài trong Box Đại học và Sau đại học


Trả lời Gởi Ðề Tài Mới
 
Ðiều Chỉnh Xếp Bài
Old 12-08-2011, 09:21 AM   #46
Nguyenhuyen_AG
+Thành Viên+
 
Nguyenhuyen_AG's Avatar
 
Tham gia ngày: Apr 2010
Bài gởi: 300
Thanks: 35
Thanked 307 Times in 151 Posts
Bài 2.1 Theo bất đẳng thức Nesbit thì

$\frac{a}{b+c}+\frac{b}{c+a}+\frac{c}{a+b}\ge \frac{3}{2} $

Nên theo bất đẳng thức Cauchy-Schwarz, ta có

$(a+b+c)\left [ \sum \frac{a}{(b+c)^2} \right ]\ge \left (\frac{a}{b+c}+\frac{b}{c+a}+\frac{c}{a+b} \right )^2\ge \frac{9}{4} $

Ngoài ra có thể chuẩn hóa cho $a+b+c=3 $ và sau đó dùng phương pháp hệ số bất định.
[RIGHT][I][B]Nguồn: MathScope.ORG[/B][/I][/RIGHT]
 
__________________
Nguyen Van Huyen
Ho Chi Minh City University of Transport
Nguyenhuyen_AG is offline   Trả Lời Với Trích Dẫn
The Following User Says Thank You to Nguyenhuyen_AG For This Useful Post:
metoan.98 (12-08-2011)
Old 12-08-2011, 09:25 AM   #47
Nguyenhuyen_AG
+Thành Viên+
 
Nguyenhuyen_AG's Avatar
 
Tham gia ngày: Apr 2010
Bài gởi: 300
Thanks: 35
Thanked 307 Times in 151 Posts
Bài 2.5 Theo bất đẳng thức Holder, ta có

$\left ( \sqrt{a}+\sqrt{b}+\sqrt{c} \right )^2\left ( a^2+b^2+c^2 \right )\ge(a+b+c)^3=27 $

Vậy để chứng minh bài toán thì ta cần chỉ ra được

$(ab+bc+ca)^2(a^2+b^2+c^2)\le 27 $

Bất đẳng thức này hiển nhiên đúng theo bất đẳng thức AM-GM
[RIGHT][I][B]Nguồn: MathScope.ORG[/B][/I][/RIGHT]
 
__________________
Nguyen Van Huyen
Ho Chi Minh City University of Transport
Nguyenhuyen_AG is offline   Trả Lời Với Trích Dẫn
The Following User Says Thank You to Nguyenhuyen_AG For This Useful Post:
metoan.98 (12-08-2011)
Old 12-08-2011, 09:40 AM   #48
Nguyenhuyen_AG
+Thành Viên+
 
Nguyenhuyen_AG's Avatar
 
Tham gia ngày: Apr 2010
Bài gởi: 300
Thanks: 35
Thanked 307 Times in 151 Posts
Bài 3.30 Bài cho hai bài toán cùng chung với nhau thì ắt hẳn chúng phải cho liên quan gì đó đến nha chứ. Sử dụng câu i) ta có thể chứng minh được câu ii) nhờ vào nhận xét "có thể giả sử $(a-1)(b-1)\ge 0 $"

Bài 5.5 Bài toán này có được nhờ vào đánh giá sau đây

$\frac{a^2}{3a^2+(b+c)^2}=\frac{a^2}{2a^2+[a^2+(b+c)^2]}\le \frac{a^2}{2a^2+2a(b+c)}=\frac{1}{2}.\frac{a}{a+b+ c} $

[RIGHT][I][B]Nguồn: MathScope.ORG[/B][/I][/RIGHT]
 
__________________
Nguyen Van Huyen
Ho Chi Minh City University of Transport
Nguyenhuyen_AG is offline   Trả Lời Với Trích Dẫn
The Following User Says Thank You to Nguyenhuyen_AG For This Useful Post:
metoan.98 (12-08-2011)
Old 12-08-2011, 09:48 AM   #49
Nguyenhuyen_AG
+Thành Viên+
 
Nguyenhuyen_AG's Avatar
 
Tham gia ngày: Apr 2010
Bài gởi: 300
Thanks: 35
Thanked 307 Times in 151 Posts
Bài 5.20 Bất đẳng thức này có thể viết lại dưới dạng

$81abc(a^2+b^2+c^2)\le (a+b+c)^5 $

hay

$abc(a^2+b^2+c^2)\le 3 $

có thể chứng minh bằng dồn biến như [Only registered and activated users can see links. ] hoặc dùng AM-GM như sau

Vì $3abc(a+b+c)\le (ab+bc+ca)^2 $ nên ta cần chứng minh

$27(ab+bc+ca)^2(a^2+b^2+c^2)\le (a+b+c)^6 $

Điều này là hiển nhiên theo AM-GM.
[RIGHT][I][B]Nguồn: MathScope.ORG[/B][/I][/RIGHT]
 
__________________
Nguyen Van Huyen
Ho Chi Minh City University of Transport
Nguyenhuyen_AG is offline   Trả Lời Với Trích Dẫn
The Following 2 Users Say Thank You to Nguyenhuyen_AG For This Useful Post:
Katyusha (12-08-2011), metoan.98 (12-08-2011)
Old 12-08-2011, 09:53 AM   #50
hungchng
Super Moderator
 
hungchng's Avatar
 
Tham gia ngày: Apr 2009
Bài gởi: 696
Thanks: 8
Thanked 800 Times in 423 Posts
Trích:
Nguyên văn bởi Nguyenhuyen_AG View Post
Về Bài 1.7 ta có một lời giải bằng Cauchy-Schwarz khá đơn giản như sau

$\begin{aligned}\left ( \sum \sqrt{a^2+3} \right )^2=\left (\sum \sqrt{a.\frac{a^2+3}{a}} \right )^2&\le (a+b+c)\left ( \frac{a^2+3}{a}+\frac{b^2+3}{b}+\frac{c^2+3}{c} \right )\\&=(a+b+c)\left [ a+b+c+3\left ( \frac{1}{a}+\frac{1}{a}+\frac{1}{a} \right ) \right ]\\&=4(a+b+c)^2\end{aligned} $

lấy căn hay vế ta sẽ có điều phải chưng minh.
Sai một chút phải là như vầy
$\begin{aligned}\left ( \sum \sqrt{a^2+3} \right )^2=\left (\sum \sqrt{a.\frac{a^2+3}{a}} \right )^2&\le (a+b+c)\left ( \frac{a^2+3}{a}+\frac{b^2+3}{b}+\frac{c^2+3}{c} \right )\\&=(a+b+c)\left [ a+b+c+3\left ( \frac{1}{a}+\frac{1}{b}+\frac{1}{c} \right ) \right ]\\&=4(a+b+c)^2\end{aligned} $
[RIGHT][I][B]Nguồn: MathScope.ORG[/B][/I][/RIGHT]
 
__________________
http://forum.mathscope.org/image.php?type=sigpic&userid=9745&dateline=1306673  632
hungchng is offline   Trả Lời Với Trích Dẫn
The Following User Says Thank You to hungchng For This Useful Post:
metoan.98 (12-08-2011)
Old 12-08-2011, 10:06 AM   #51
Nguyenhuyen_AG
+Thành Viên+
 
Nguyenhuyen_AG's Avatar
 
Tham gia ngày: Apr 2010
Bài gởi: 300
Thanks: 35
Thanked 307 Times in 151 Posts
Trích:
Nguyên văn bởi hungchng View Post
Sai một chút phải là như vầy
$\begin{aligned}\left ( \sum \sqrt{a^2+3} \right )^2=\left (\sum \sqrt{a.\frac{a^2+3}{a}} \right )^2&\le (a+b+c)\left ( \frac{a^2+3}{a}+\frac{b^2+3}{b}+\frac{c^2+3}{c} \right )\\&=(a+b+c)\left [ a+b+c+3\left ( \frac{1}{a}+\frac{1}{b}+\frac{1}{c} \right ) \right ]\\&=4(a+b+c)^2\end{aligned} $
Thầy ơi hai lời giải giống nhau đâu khác gì đâu thầy.
[RIGHT][I][B]Nguồn: MathScope.ORG[/B][/I][/RIGHT]
 
__________________
Nguyen Van Huyen
Ho Chi Minh City University of Transport
Nguyenhuyen_AG is offline   Trả Lời Với Trích Dẫn
The Following User Says Thank You to Nguyenhuyen_AG For This Useful Post:
metoan.98 (12-08-2011)
Old 12-08-2011, 10:29 AM   #52
hungchng
Super Moderator
 
hungchng's Avatar
 
Tham gia ngày: Apr 2009
Bài gởi: 696
Thanks: 8
Thanked 800 Times in 423 Posts
Trích:
Nguyên văn bởi Nguyenhuyen_AG View Post
Thầy ơi hai lời giải giống nhau đâu khác gì đâu thầy.
Lộn một chút chỗ $ 3\left(\frac{1}{a}+\frac{1}{a}+\frac{1}{a}\right) $
[RIGHT][I][B]Nguồn: MathScope.ORG[/B][/I][/RIGHT]
 
__________________
http://forum.mathscope.org/image.php?type=sigpic&userid=9745&dateline=1306673  632
hungchng is offline   Trả Lời Với Trích Dẫn
The Following User Says Thank You to hungchng For This Useful Post:
metoan.98 (12-08-2011)
Old 19-09-2011, 09:50 PM   #53
Super Dốt
+Thành Viên+
 
Super Dốt's Avatar
 
Tham gia ngày: Sep 2011
Bài gởi: 5
Thanks: 0
Thanked 0 Times in 0 Posts
Cảm ơn BQT rất nhiều.
Mong chờ các tuyển tập khác của diễn đàn
[RIGHT][I][B]Nguồn: MathScope.ORG[/B][/I][/RIGHT]
 
__________________
Super Dốt is offline   Trả Lời Với Trích Dẫn
Old 19-09-2011, 10:49 PM   #54
phamtoan
Banned
 
Tham gia ngày: Apr 2011
Đến từ: VMF
Bài gởi: 313
Thanks: 266
Thanked 63 Times in 50 Posts
Gửi tin nhắn qua Yahoo chát tới phamtoan
Xin cảm ơn mọi người về cuốn tuyển tập bổ ích. Hi vọng cuốn sau sẽ được cải thiện hơn. Một tài liệu tuyệt vời.
[RIGHT][I][B]Nguồn: MathScope.ORG[/B][/I][/RIGHT]
 
phamtoan is offline   Trả Lời Với Trích Dẫn
Old 09-10-2011, 12:10 PM   #55
truongvoki_bn
+Thành Viên Danh Dự+
 
truongvoki_bn's Avatar
 
Tham gia ngày: Oct 2009
Đến từ: _chuyenbacninh_
Bài gởi: 614
Thanks: 72
Thanked 539 Times in 208 Posts
Đúng là lên trường bị cô lập với máy tính lên không biết MS đã xuất xưởng cái này. Để khi nào làm cái Topic BĐT 2 thì mình xí xớn cái nhể
p/s: Chưa học latex @@
Oạch có cả một em Chuyên Bắc Ninh tham gia nữa à
[RIGHT][I][B]Nguồn: MathScope.ORG[/B][/I][/RIGHT]
 
__________________
Cuộc sống là không chờ đợi


Đại học thôi. Lăn tăn gì nữa
truongvoki_bn is offline   Trả Lời Với Trích Dẫn
The Following User Says Thank You to truongvoki_bn For This Useful Post:
n.v.thanh (09-10-2011)
Old 09-10-2011, 12:29 PM   #56
n.v.thanh
Moderator
 
n.v.thanh's Avatar
 
Tham gia ngày: Nov 2009
Bài gởi: 2,849
Thanks: 2,980
Thanked 2,537 Times in 1,008 Posts
Em là em kết nhất quả bìa đẹp rạng ngời mà chói lòa cả mũi
[RIGHT][I][B]Nguồn: MathScope.ORG[/B][/I][/RIGHT]
 
n.v.thanh is offline   Trả Lời Với Trích Dẫn
The Following User Says Thank You to n.v.thanh For This Useful Post:
batigoal (09-10-2011)
Old 23-10-2011, 10:14 PM   #57
5434
+Thành Viên+
 
5434's Avatar
 
Tham gia ngày: Sep 2011
Đến từ: no*i ty bă't đâ'u
Bài gởi: 695
Thanks: 121
Thanked 335 Times in 214 Posts
Bài 1.1 sai giả thiết thì phải
[RIGHT][I][B]Nguồn: MathScope.ORG[/B][/I][/RIGHT]
 
__________________

5434 is offline   Trả Lời Với Trích Dẫn
Old 04-11-2011, 03:29 PM   #58
thuonghd
+Thành Viên+
 
Tham gia ngày: Mar 2009
Bài gởi: 2
Thanks: 2
Thanked 0 Times in 0 Posts
Trích:
Nguyên văn bởi batigoal View Post
Sách thì nên để tên thật em à, cho nick bên cạnh tên trường, Tỉnh (TP) không đẹp lắm.
Cụ thể nick của các anh em trong nhóm

1. Đào Thái Hiệp( Kratos )
2. Phạm Tiến Kha( Ptk_1411)
3. Phạm Tuấn Huy ( Pth_Tdn )
4. Phạm Quang Hưng( Ladykillah96 )
5 Tăng Hải Tuân( Liltee. )
6. Nguyễn Văn Khánh ( Nvm )
7 Lê Đức Cảnh.( Crazy Nhoc )
8. Mạc Đức Trí( Asdfghj )
9 Hoàng Minh Quân( Batigoal )
10.Nguyễn Thị Nguyên Khoa(Liverpool29)
Thật tuyệt vời. Cảm on các bạn rất nhiều. Một quyển sách nhiều thú vị.
[RIGHT][I][B]Nguồn: MathScope.ORG[/B][/I][/RIGHT]
 
thuonghd is offline   Trả Lời Với Trích Dẫn
Old 28-03-2012, 10:16 PM   #59
AnhIsGod
+Thành Viên+
 
Tham gia ngày: Nov 2011
Đến từ: Vô cực
Bài gởi: 267
Thanks: 358
Thanked 48 Times in 32 Posts
Hình như bài 1.11 làm sai rồi.
[RIGHT][I][B]Nguồn: MathScope.ORG[/B][/I][/RIGHT]
 
AnhIsGod is offline   Trả Lời Với Trích Dẫn
Old 29-10-2012, 11:53 PM   #60
quangtruong97
+Thành Viên+
 
Tham gia ngày: Sep 2012
Bài gởi: 10
Thanks: 7
Thanked 1 Time in 1 Post
Trang 6, đoạn biểu diễn bất đẳng thức CS thiếu bình phương anh ơi
Bài 1.1 trang 10 tại sao $ x+y+z=1 $ mà $ 2^{x+y+z}=64 $
[RIGHT][I][B]Nguồn: MathScope.ORG[/B][/I][/RIGHT]
 
quangtruong97 is offline   Trả Lời Với Trích Dẫn
Trả lời Gởi Ðề Tài Mới

Bookmarks


Quuyền Hạn Của Bạn
You may not post new threads
You may not post replies
You may not post attachments
You may not edit your posts

BB code is Mở
Smilies đang Mở
[IMG] đang Mở
HTML đang Tắt

Chuyển đến


Múi giờ GMT. Hiện tại là 02:36 AM.


Powered by: vBulletin Copyright ©2000-2024, Jelsoft Enterprises Ltd.
Inactive Reminders By mathscope.org
[page compression: 102.06 k/118.08 k (13.57%)]